Mathcenter Forum  

Go Back   Mathcenter Forum > คณิตศาสตร์โอลิมปิก และอุดมศึกษา > อสมการ
สมัครสมาชิก คู่มือการใช้ รายชื่อสมาชิก ปฏิทิน ข้อความวันนี้

ตั้งหัวข้อใหม่ Reply
 
เครื่องมือของหัวข้อ ค้นหาในหัวข้อนี้
  #1  
Old 22 สิงหาคม 2007, 21:23
dektep's Avatar
dektep dektep ไม่อยู่ในระบบ
กระบี่ประสานใจ
 
วันที่สมัครสมาชิก: 07 มีนาคม 2007
ข้อความ: 580
dektep is on a distinguished road
Default hard inequalities

1.Let a,b,c,d be positeve numbers such that abcd=1 Prove that
$\frac{1}{1+a+a^2+a^3}$+$\frac{1}{1+b+b^2+b^3}$+$\frac{1}{1+c+c^2+c^3}$+$\frac{1}{1+d+d^2+d^3}$ $\geq$ 1
2.Let a,b,c be non-negative numbers, no two of which are zero.Prove that
$\frac{a^4}{a^3+b^3}$+$\frac{b^4}{b^3+c^3}$+$\frac{c^4}{c^3+a^3}$ $\geq$ $\frac{a+b+c}{2}$
ตอบพร้อมอ้างอิงข้อความนี้
  #2  
Old 22 สิงหาคม 2007, 21:30
dektep's Avatar
dektep dektep ไม่อยู่ในระบบ
กระบี่ประสานใจ
 
วันที่สมัครสมาชิก: 07 มีนาคม 2007
ข้อความ: 580
dektep is on a distinguished road
Default

3.Show that for positive reals a,b,c
$$\sum_{cyc}^{} \frac{1}{(4a^2-ab+4b^2)} ≥ \frac{9}{7(a^2+b^2+c^2)}$$

28 สิงหาคม 2007 18:41 : ข้อความนี้ถูกแก้ไขแล้ว 2 ครั้ง, ครั้งล่าสุดโดยคุณ dektep
เหตุผล: แก้โคด
ตอบพร้อมอ้างอิงข้อความนี้
  #3  
Old 25 สิงหาคม 2007, 20:50
putmusic putmusic ไม่อยู่ในระบบ
จอมยุทธ์หน้าหยก
 
วันที่สมัครสมาชิก: 11 สิงหาคม 2007
ข้อความ: 183
putmusic is on a distinguished road
Default

อ่านไม่ออกเลยครับ ภ้าแปลเป็นไทย ผมยังพอทำได้บ้างนะครับ แต่อันนี้เป็นอังกฤา ผมคงหมดหวังครับ
ตอบพร้อมอ้างอิงข้อความนี้
  #4  
Old 26 สิงหาคม 2007, 22:42
Spotanus's Avatar
Spotanus Spotanus ไม่อยู่ในระบบ
จอมยุทธ์หน้าหยก
 
วันที่สมัครสมาชิก: 28 มีนาคม 2007
ข้อความ: 171
Spotanus is on a distinguished road
Default

ข้อหนึ่งผมเดาว่าน่าจะใช้ จัดรูป แล้ว AM-GM ครับ
เดี๋ยวจะเอา Solution มาลงให้ (ถ้าออกนะครับ)
ตอบพร้อมอ้างอิงข้อความนี้
  #5  
Old 27 สิงหาคม 2007, 19:14
kanakon's Avatar
kanakon kanakon ไม่อยู่ในระบบ
บัณฑิตฟ้า
 
วันที่สมัครสมาชิก: 31 ตุลาคม 2006
ข้อความ: 523
kanakon is on a distinguished road
Default

ข้อสามคุณ dektep ลองใช้ $ สองตัว(ตามใค้ด)จะสวยกว่านะครับ
อ้างอิง:
ข้อความเดิมเขียนโดยคุณ dektep View Post
3.Show that for positive reals a,b,c
$$\sum_{cyc}^{} \frac{1}{(4a^2-ab+4b^2)} ≥ \frac{9}{7(a^2+b^2+c^2)}$$
__________________
ค ว า ม รั บ ผิ ด ช อ บ

$$|I-U|\rightarrow \infty $$

27 สิงหาคม 2007 19:17 : ข้อความนี้ถูกแก้ไขแล้ว 2 ครั้ง, ครั้งล่าสุดโดยคุณ kanakon
ตอบพร้อมอ้างอิงข้อความนี้
  #6  
Old 07 ธันวาคม 2007, 13:55
kanakon's Avatar
kanakon kanakon ไม่อยู่ในระบบ
บัณฑิตฟ้า
 
วันที่สมัครสมาชิก: 31 ตุลาคม 2006
ข้อความ: 523
kanakon is on a distinguished road
Default

อ้างอิง:
ข้อความเดิมเขียนโดยคุณ dektep View Post
1.Let a,b,c,d be positeve numbers such that abcd=1 Prove that
$\frac{1}{1+a+a^2+a^3}$+$\frac{1}{1+b+b^2+b^3}$+$\frac{1}{1+c+c^2+c^3}$+$\frac{1}{1+d+d^2+d^3}$ $\geq$ 1
A.M.-H.M. inequality
$$\frac{\frac{1}{1+a+a^2+a^3}+\frac{1}{1+b+b^2+b^3}+\frac{1}{1+c+c^2+c^3}+\frac{1}{1+d+d^2+d^3}}{4}\geq \frac{4}{4+(a+b+c+d)+(a^2+b^2+c^2+d^2)+(a^3+b^3+c^3+d^3)} $$
$$\because a+b+c+d\geq 4,a^2+b^2+c^2+d^2\geq 4,a^3+b^3+c^3+d^3\geq 4$$
$$\therefore \frac{1}{1+a+a^2+a^3}+\frac{1}{1+b+b^2+b^3}+\frac{1}{1+c+c^2+c^3}+\frac{1}{1+d+d^2+d^3}\geq \frac{16}{16}=1 $$
__________________
ค ว า ม รั บ ผิ ด ช อ บ

$$|I-U|\rightarrow \infty $$

07 ธันวาคม 2007 14:38 : ข้อความนี้ถูกแก้ไขแล้ว 1 ครั้ง, ครั้งล่าสุดโดยคุณ kanakon
เหตุผล: h.m.
ตอบพร้อมอ้างอิงข้อความนี้
  #7  
Old 07 ธันวาคม 2007, 16:36
dektep's Avatar
dektep dektep ไม่อยู่ในระบบ
กระบี่ประสานใจ
 
วันที่สมัครสมาชิก: 07 มีนาคม 2007
ข้อความ: 580
dektep is on a distinguished road
Default

อ้างอิง:
ข้อความเดิมเขียนโดยคุณ kanakon View Post
A.M.-H.M. inequality
$$\frac{\frac{1}{1+a+a^2+a^3}+\frac{1}{1+b+b^2+b^3}+\frac{1}{1+c+c^2+c^3}+\frac{1}{1+d+d^2+d^3}}{4}\geq \frac{4}{4+(a+b+c+d)+(a^2+b^2+c^2+d^2)+(a^3+b^3+c^3+d^3)} $$
$$\because a+b+c+d\geq 4,a^2+b^2+c^2+d^2\geq 4,a^3+b^3+c^3+d^3\geq 4$$
$$\therefore \frac{1}{1+a+a^2+a^3}+\frac{1}{1+b+b^2+b^3}+\frac{1}{1+c+c^2+c^3}+\frac{1}{1+d+d^2+d^3}\geq \frac{16}{16}=1 $$
ผมคิดว่าตรงสุดท้ายน่าจะกลับข้างครับ เพราะว่า ถ้า $a+b+c+d\geq 4,a^2+b^2+c^2+d^2\geq 4,a^3+b^3+c^3+d^3\geq 4$
แล้ว $$\frac{4}{4+(a+b+c+d)+(a^2+b^2+c^2+d^2)+(a^3+b^3+c^3+d^3)} \leq \frac{1}{4}$$

07 ธันวาคม 2007 16:37 : ข้อความนี้ถูกแก้ไขแล้ว 2 ครั้ง, ครั้งล่าสุดโดยคุณ dektep
ตอบพร้อมอ้างอิงข้อความนี้
ตั้งหัวข้อใหม่ Reply


หัวข้อคล้ายคลึงกัน
หัวข้อ ผู้ตั้งหัวข้อ ห้อง คำตอบ ข้อความล่าสุด
Hard Inequalities from Mathlinks Contest gools อสมการ 1 11 ธันวาคม 2005 06:46
Not really hard questions from Germany: Part1 nongtum ปัญหาคณิตศาสตร์ทั่วไป 10 09 พฤษภาคม 2005 08:28
A very hard inequality Punk อสมการ 13 17 เมษายน 2005 01:39


กฎการส่งข้อความ
คุณ ไม่สามารถ ตั้งหัวข้อใหม่ได้
คุณ ไม่สามารถ ตอบหัวข้อได้
คุณ ไม่สามารถ แนบไฟล์และเอกสารได้
คุณ ไม่สามารถ แก้ไขข้อความของคุณเองได้

vB code is On
Smilies are On
[IMG] code is On
HTML code is Off
ทางลัดสู่ห้อง


เวลาที่แสดงทั้งหมด เป็นเวลาที่ประเทศไทย (GMT +7) ขณะนี้เป็นเวลา 09:08


Powered by vBulletin® Copyright ©2000 - 2024, Jelsoft Enterprises Ltd.
Modified by Jetsada Karnpracha